1. Introduction

All the groups considered in the paper are finite, and \(G\) always denotes a finite group. Moreover, \(\mathbb{P}\) is the set of all primes, and \(\sigma\) is some partition of \(\mathbb{P}\), i.e.,

$$\sigma=\{\sigma_{i} \mid i \in I\},$$

where \(\mathbb{P}=\bigcup_{i \in I} \sigma_{i}\) and \(\sigma_{i} \cap \sigma_{j}=\varnothing\) for all \( i \ne j\). The symbol \(\pi(G)\) denotes the set of all prime divisors of \(|G|\), and \(\sigma(G)=\{\sigma_{i}\mid\sigma_{i}\cap \pi(G)\ne\varnothing\}\). We say that a chain of subgroups

$$\cdots<M_{3}<M_{2}<M_{1}<M_{0}=G$$

of \(G\) is a maximal chain in \(G\) if \(M_{i}\) is a maximal subgroup in \(M_{i-1}\) for all \(i\). The groups \(A\) and \(B\) are called groups of equal order if \(|A|=|B|\). If \(A\) is a subgroup of a group \(G\), then \(A_{G}\) is the largest normal subgroup of \(G\) contained in \(A\).

Let \(\mathfrak{F}\) be a class of groups. Then a subgroup \(A\) of a group \(G\) is said to be \(\mathfrak{F}\)-subnormal in the sense of Kegel [1] or \(K\)-\(\mathfrak{F}\)-subnormal in \(G\) [2] if \(G\) contains a series of subgroups

$$A=A_{0} \le A_{1} \le \cdots \le A_{t}=G$$

such that either \(A_{i-1} \trianglelefteq A_{i}\) or \(A_{i}/(A_{i-1})_{A_{i}} \in \mathfrak{F}\) for all \(i=1,\dots,t\).

Recall some notions of the theory of \(\sigma\)-properties of a group [3]–[6].

A group \(G\) is said to be \(\sigma\)-primary if \(G\) is a \(\sigma_{i}\)-group for some \(i=i(G)\); \(\sigma\)-solvable if every chief factor of \(G\) is \(\sigma\)-primary. We use the symbols \(\mathfrak{S}_{\sigma}\) and \(\mathfrak{S}\) to denote the classes of all \(\sigma\)-solvable and all solvable groups, respectively; \(G^{\mathfrak{S}_{\sigma}}\) stands for the intersection of all normal subgroups \(N\) of a group \(G\) with \(G/N\in \mathfrak{S}_{\sigma}\).

A set of subgroups \(\mathcal{H}\) of a group \(G\) is called a complete Hall \(\sigma\)-set of \(G\) if every element \(\ne 1\) of the set \(\mathcal{H}\) is a Hall \(\sigma_{i}\)-subgroup of \(G\) for some \(i\) and \(\mathcal{H}\) contains exactly one Hall \(\sigma_{i}\)-subgroup of \(G\) for every \(\sigma_{i}\in \sigma(G)\).

Definition 1.

We say that a subgroup \(A\) of a group \(G\) is weakly \(K\)-\(\mathfrak{S}_{\sigma}\)-subnormal in \(G\) if there are \(K\)-\(\mathfrak{S}_{\sigma}\)-subnormal subgroups \(T\) and \(S\) such that \(G=AT\) and \(A \cap T \le S \le A\).

Remark 1.

(i) Each \(K\)-\(\mathfrak{S}_{\sigma}\)-subnormal subgroup \(S=S \cap G\) is weakly \(K\)-\(\mathfrak{S}_{\sigma}\)-subnormal in the group \(G\), since \(G\) is a \(K\)-\(\mathfrak{S}_{\sigma}\)-subnormal subgroup of \(G\).

(ii) A subgroup \(A\) of a group \(G\) is said to be \(\sigma\)-subnormal in \(G\) [3] if there is a series of subgroups \(A=A_{0} \le A_{1} \le \cdots A_{n}=G\) such that either \(A_{i-1} \trianglelefteq A_{i}\) or the quotient \(A_{i}/(A_{i-1})_{A_{i}}\) is \(\sigma\)-primary for all \(i=1,\dots,n\). Every subnormal subgroup is \(\sigma\)-subnormal, and every \(\sigma\)-subnormal subgroup is \(K\)-\(\mathfrak{S}_{\sigma}\)-subnormal in the group.

(iii) A subgroup \(S\) of a group \(G\) is said to be \(\sigma\)-permutable in \(G\) [3] if \(G\) has a complete Hall \(\sigma\)-set \(\mathcal H\) such that \(SH^{x}=H^{x}S\) for all \(H\in \mathcal{H}\) and all \(x\in G\).

By Theorem B of [3], every \(\sigma\)-permutable subgroup is \(\sigma\)-subnormal and, therefore, \(K\)-\(\mathfrak{S}_{\sigma}\)-subnormal in the group.

Now consider the following example.

Example 1.

(i) A subgroup \(A\) of a group \(G\) is said to be weakly \(\sigma\)-permutable in \(G\) ([7], [8]) if \(G\) contains a \(\sigma\)-permutable subgroup \(S\) and a \(\sigma\)-subnormal subgroup \(T\) such that \(G=AT\) and \(A \cap T \le S \le A\). By Remark 1 (ii, iii), every weakly \(\sigma\)-permutable subgroup is weakly \(K\)-\(\mathfrak{S}_{\sigma}\)-subnormal in the group.

(ii) In the general case, the class of all weakly \(K\)-\(\mathfrak{S}_{\sigma}\)-subnormal subgroups is wider than the class of all weakly \(\sigma\)-permutable subgroups. Let \(\sigma=\{\{2\},\{3\},\dots\}\). In this case, every \(\sigma\)-subnormal subgroup is subnormal, and any \(\sigma\)-permutable subgroup \(H\) is \(S\)-permutable in the group ([9], [10]), i.e., \(HP=PH\) for all Sylow subgroups \(P\) of the group.

Now let \(A\) be a non-Abelian group of order \(p^{3}\) of a simple odd exponent \(p\). Let \(G=A \wr C_{2}=B \rtimes C_{2}\), where \(B\) is the base of the regular wreath product \(G\). Let \(L=\Phi(A_{1})\), where \(A_{1}\) is the first copy of the group \(A\) in \(B\). Then \(|L|=p\), \(L \le \Phi(B)\), and \(L\) is subnormal, and hence weakly \(K\)-\(\mathfrak{S}\)-subnormal in \(G\), considering Remark 1.2 (ii). It is also clear that \(L\) is a nonnormal subgroup of \(G\). Now let us show that \(L\) is not weakly \(S\)-permutable in \(G\). Suppose that \(G\) contains a subnormal subgroup \(T\) and an \(S\)-permutable subgroup \(S\) such that \(G=LT\) and \(L \cap T \le S \le L\). Then \(B=L(T \cap B)\), and thus \(T\cap B=B\), since \(L \le \Phi(B)\). Hence \(L \cap T=L=S\) is \(S\)-permutable in \(G\). Then \(L\) is normal in \(G\) by [9, Lemma 1.2.16]. This contradiction shows that \(L\) is not weakly \(S\)-permutable in \(G\).

(iii) A subgroup \(A\) of a group \(G\) is said to be \(c\)-normal in \(G\) [11] if \(AT=G\) and \(A \cap T \le A_{G}\) for some normal subgroup \(T\) of \(G\). Consequently, any \(c\)-normal subgroup is weakly \(K\)-\(\mathfrak{S}_{\sigma}\)-subnormal in the group for every partition \(\sigma\) of \(\mathbb{P}\).

(iv) A subgroup \(M\) of a group \(G\) is said to be

  1. (a)

    modular in \(G\) [12] if \(M\) is a modular element (in the sense of Kurosh [12, p. 43]) of the lattice of all subgroups \(\mathcal{L}(G)\) of \(G\), i.e., the following conditions hold:

    1. (1)

      \(\langle X,M \cap Z \rangle=\langle X,M \rangle \cap Z\) for all \(X \le G\) and \(Z \le G\) such that \(X \le Z\);

    2. (2)

      \(\langle M,Y \cap Z \rangle=\langle M,Y \rangle \cap Z\) for all \(Y \le G\) and \(Z \le G\) such that \(M \le Z\);

  2. (b)

    submodular in \(G\) ([13], [14]) if \(G\) contains a series of subgroups \(A=A_{0} \le A_{1} \le\cdots\le A_{t}=G\) such that \(A_{i-1}\) is a modular subgroup of \(A_{i}\) for all \(i=1,\dots,t\).

It follows from the main result of the theory of modular subgroups [12, Theorem 5.1.14] that every submodular subgroup is \(K\)-\(\mathfrak{S}\)-subnormal and, therefore, weakly \(K\)-\(\mathfrak{S}\)-subnormal in the group.

Although the concept of \(\sigma\)-solvable group was first introduced in a recent paper [3], this concept proved to be very useful in the analysis of many open questions (see, for example, [3]–[8], [15]–[26]), and thus the problem of finding conditions under which a group is \(\sigma\)-solvable is very interesting and relevant. In this paper, we prove the following result.

Theorem 1.

The following conditions are equivalent:

  1. (i)

    a group \(G\) is \(\sigma\) -solvable;

  2. (ii)

    every subgroup of \(G\) is \(K\) - \(\mathfrak{S}_{\sigma}\) -subnormal;

  3. (iii)

    \(G\) has a complete Hall \(\sigma\) -set \(\mathcal H\) all of whose elements are weakly \(K\) - \(\mathfrak{S}_{\sigma}\) -subnormal in \(G\) ;

  4. (iv)

    in every maximal chain \(\cdots < M_{3} < M_{2} < M_{1} < M_{0}=G\) of \(G\) , at least one of the subgroups \(M_{3}\) , \(M_{2}\) or \(M_{1}\) is weakly \(K\) - \(\mathfrak{S}_{\sigma}\) -subnormal in \(G\) ;

  5. (v)

    \(i_{K,\mathfrak{S}_{\sigma}}(G)\le 2|\sigma(G)|\) , where \(i_{K,\mathfrak{S}_{\sigma}}(G)\) is the number of classes of non- \(K\) - \(\mathfrak{S}_{\sigma}\) -subnormal subgroups of \(G\) of equal order.

Theorem 1 covers many known results. In particular, taking into account Remark 1 and Example 1, we see that the following well-known results are special cases of this theorem.

Corollary 1 (Guo, Skiba [19]).

If, in every maximal chain \(\cdots<M_{3}<M_{2}<M_{1}<M_{0}=G\) of a group \(G\) , at least one of the subgroups \(M_{3}\) , \(M_{2}\) , or \(M_{1}\) is \(\sigma\) -subnormal in \(G\) , then \(G\) is \(\sigma\) -solvable.

Corollary 2 (Zhang, Wu, Guo [7]).

If \(G\) has a complete Hall \(\sigma\) -set \(\mathcal H\) all of whose elements are weakly \(\sigma\) -permutable in \(G\) , then \(G\) is \(\sigma\) -solvable.

Corollary 3 (Zimmermann [14]).

If, in every maximal chain \(\cdots<M_{3}<M_{2}<M_{1}<M_{0}=G\) of a group \(G\) , at least one of the subgroups \(M_{3}\) , \(M_{2}\) , or \(M_{1}\) is submodular in \(G\) , then \(G\) is solvable.

Corollary 4 (Spencer [27]).

If, in every maximal chain \(\cdots<M_{3}<M_{2}<M_{1}<M_{0}=G\) of a group \(G\) , at least one of the subgroups \(M_{3}\) , \(M_{2}\) , or \(M_{1}\) is subnormal in \(G\) , then \(G\) is solvable.

Corollary 5 (Schmid [28]).

A group \(G\) is solvable if each of its 3-maximal subgroups is modular.

Corollary 6 (Kovaleva [15]).

If \(i_{\sigma}(G) \le 2|\sigma(G)|\) , where \(i_{\sigma}(G)\) stands for the number of classes of non- \({\sigma}\) -subnormal subgroups of equal order of a group \(G\) , then \(G\) is \(\sigma\) -solvable.

Corollary 7 (Lu, Meng [29]).

If the number of conjugacy classes of nonsubnormal subgroups of a group \(G\) does not exceed \(2|\pi(G)|\) , then \(G\) is solvable.

2. Proof of Theorem 1

Lemma 1.

(1) The class \(\mathfrak{S}_{\sigma}\) is closed with respect to taking products of normal \(\mathfrak{S}_{\sigma}\)-subgroups, homomorphic images, and subgroups. Moreover, any extension of a \(\sigma\)-solvable group with the help of a \(\sigma\)-solvable group is \(\sigma\)-solvable.

(2) A group is \(\sigma\)-solvable if and only if all its maximal subgroups are \(K\)-\(\mathfrak{S}_{\sigma}\)-subnormal.

Proof.

(1) This assertion is obvious.

(2) It is clear that, in every \(\sigma\)-solvable group, all its maximal subgroups are \(K\)-\(\mathfrak{S}_{\sigma}\)-subnormal.

Suppose now that all maximal subgroups of a group \(G\) are \(K\)-\(\mathfrak{S}_{\sigma}\)-subnormal in \(G\). Then \(G/M_{G}\in \mathfrak{S}_{\sigma}\) for every maximal subgroup \(M\) of \(G\). Therefore, \(G/\Phi(G) \in \mathfrak{S}_{\sigma}\) and, consequently, \(G \in \mathfrak{S}_{\sigma}\), by part (1).

This completes the proof of the lemma.

From Lemma 1 and from the main result of [1], we obtain the following assertion.

Lemma 2.

The set of all \(K\) - \(\mathfrak{S}_{\sigma}\) -subnormal subgroups of a group \(G\) forms a sublattice of the lattice of all subgroups of \(G\) .

Lemma 3.

Let \(A\) , \(H\) , and \(N\) be subgroups of \(G\) , where \(A\) is \(K\) - \(\mathfrak{S}_{\sigma}\) -subnormal and \(N\) is normal in \(G\) . Then

  1. (1)

    \(A\cap H\) is \(K\) - \(\mathfrak{S}_{\sigma}\) -subnormal in \(H\) ;

  2. (2)

    \(AN/N\) is \(K\) - \(\mathfrak{S}_{\sigma}\) -subnormal in \(G/N\) ;

  3. (3)

    if \(N\le H\) and \(H/N\) is \(K\) - \(\mathfrak{S}_{\sigma}\) -subnormal in \(G/N\) , then \(H\) is \(K\) - \(\mathfrak{S}_{\sigma}\) -subnormal in \(G\) ;

  4. (4)

    the subgroup \(A^{\mathfrak{S}_{\sigma}}\) is subnormal in \(G\) ;

  5. (5)

    if \(A\) is \(\sigma\) -solvable and \(N\) is a non- \(\sigma\) -primary minimal normal subgroup of \(G\) , then \(A\le C_{G}(N)\) .

Proof.

(1)–(4). These assertions are corollaries of Lemmas 6.1.6, 6.1.7, and 6.1.9 in [2].

(5) Assume that this assertion fails to hold; let \(G\) be a counterexample of minimal order. Then \(A\ne G\). By assumption, there is a series of subgroups

$$A=A_{0} \le A_{1} \le \cdots \le A_{r}=G$$

such that either \(A_{i-1} \trianglelefteq A_{i}\) or \(A_{i}/(A_{i-1})_{A_{i}}\) is \(\sigma \)-solvable for all \(i=1,\dots,r\). Let \(M=A_{r-1}\). Without loss of generality, we can assume that \(M\ne G\).

Let \(E=NA\). Let us first assume that \( E < G\). It is clear that \(N=N_{1}\times \cdots \times N_{n}\) for some minimal normal subgroups \(N_{1},\dots,N_{n}\) of \(E\) and \(N_{i}\) is not \(\sigma\)-primary for all \(i\). By Lemma 3 (1), \(A\) is \(K\)-\(\mathfrak{S}_{\sigma}\)-subnormal in \(E\) and, therefore, due to the choice of \(G\), this means that \(A \le C_{E}(N_{i})\) for all \(i\) and, therefore, \(A \le C_{E}(N)\). Hence \(NA=E=G\). Then \(N\nleq M\), and hence \(G/M_{G}\) is not \(\sigma\)-solvable, since \(N\simeq NM_{G}/M_{G}\) is not \(\sigma\)-primary. This implies that \(M\) is normal in \(G\), and hence \(N\cap M=1\). This implies that \([N,M]=1\), and thus \(A\le C_{G}(N)\).

This completes the proof of the lemma.

Lemma 4.

Let \(A\) , \(H\) , and \(N\) be subgroups of a group \(G\) , where \(A\) is weakly \(K\) - \(\mathfrak{S}_{\sigma}\) -subnormal and \(N\) is normal in \(G\) .

  1. (1)

    If either \(N\le A\) or \((|N|,|A|)=1\) , then \(AN/N\) is weakly \(K\) - \(\mathfrak{S}_{\sigma}\) -subnormal in \(G/N\) .

  2. (2)

    If \(N \le H\) and \(H/N\) is weakly \(K\) - \(\mathfrak{S}_{\sigma}\) -subnormal in \(G/N\) , then \(H\) is weakly \(K\) - \(\mathfrak{S}_{\sigma}\) -subnormal in \(G\) .

  3. (3)

    If \(A \le E \le G\) , then \(A\) is weakly \(K\) - \(\mathfrak{S}_{\sigma}\) -subnormal in \(E\) .

Proof.

Let \(T\) and \(S\) be \(K\)-\(\mathfrak{S}_{\sigma}\)-subnormal subgroups of \(G\) such that \(G=AT\) and \(A \cap T \le S \le A\).

(1) First, note that

$$(AN/N)(TN/N)=ATN/N=G/N,\qquad SN/N\le AN/N,$$

where the subgroups \(TN/N\) and \(SN/N\) are \(K\)-\(\mathfrak{S}_{\sigma}\)-subnormal in \(G/N\) by Lemma 3 (2). Thus, it remains only to show that

$$(AN/N) \cap (TN/N) \le SN/N.$$

If \(N \le A\), then

$$(AN/N) \cap (TN/N)=(A \cap TN)/N=N(A \cap T)/N \le SN/N.$$

Suppose now that \((|N|,|A|)=1\). Since \(G=AT\), it follows that \(|G:T|\) divides \(|A|\). Hence \(|NT:T|\) divides \(|A|\). However, \(|NT:T|=|N: N \cap T|\) divides \(|N|\). Thus, \(N \le T\), and hence

$$(AN/N) \cap (TN/N)=(AN/N) \cap (T/N)=(AN \cap T)/N= N(A \cap T)/N \le SN/N.$$

Therefore, the subgroup \(AN/N\) is weakly \(K\)-\(\mathfrak{S}_{\sigma}\)-subnormal in \(G/N\).

(2) By assumption, \(G/N\) contains \(K\)-\(\mathfrak{S}_{\sigma}\)-subnormal subgroups \(Z/N\) and \(D/N\) such that

$$G/N=(H/N)(Z/N),\qquad (H\cap Z)/N=(H/N) \cap (Z/N) \le D/N \le H/N.$$

Then \(G=HZ\) and \(H \cap Z \le D \le H\), where \(Z\) and \(D\) are \(K\)-\(\mathfrak{S}_{\sigma}\)-subnormal subgroups of \(G\) by Lemma 3 (3) and, therefore, \(H\) is weakly \(K\)-\(\mathfrak{S}_{\sigma}\)-subnormal in \(G\).

(3) Note that

$$E=E \cap AT=A(E \cap T),\qquad A \cap (E \cap T)=A \cap T \le S \le A,$$

where \(E \cap T\) and \(S\) are weakly \(K\)-\(\mathfrak{S}_{\sigma}\)-subnormal in \(E\) by Lemma 3 (1). Hence \(A\) is weakly \(K\)-\(\mathfrak{S}_{\sigma}\)-subnormal in \(E\).

This completes the proof of the lemma.

Proof of Theorem 1.

(i) \(\Rightarrow\) (ii) Let \(H\le M < G\), where \(M\) is a maximal subgroup of \(G\). Then \(M\) is \(K\)-\(\mathfrak{S}_{\sigma}\)-subnormal in \(G\) by Lemma 1 (2). On the other hand, \(H\) is \(K\)-\(\mathfrak{S}_{\sigma}\)-subnormal in \(M\) by induction. Hence \(H\) is \(K\)-\(\mathfrak{S}_{\sigma}\)-subnormal in \(G\).

The implication (ii) \(\Rightarrow\) (i) follows from Lemma 1 (2).

Since a \(\sigma\)-solvable group is \(\sigma_{i}\)-separable, and hence has a Hall \(\sigma_{i}\)-subgroup for all \(i\), it follows from the implication (ii) \(\Rightarrow\) (i) and from Remark 1 (i) that (ii) \(\Rightarrow\) (iii), (iv), (v).

(iii) \(\Rightarrow\) (i) Suppose that this assertion fails to hold; let \(G\) be a counterexample of minimal order. Let \(\mathcal{H}=\{H_{1},\dots,H_{t}\}\). Then \(t > 1\). Without loss of generality, we can assume that \(H_{i}\) is a nonidentity \(\sigma_{i}\)-group for all \(i=1,\dots,t\). According to the condition, for any \(i\), the group \(G\) has \(K\)-\(\mathfrak{S}_{\sigma}\)-subnormal subgroups \(T_{i}\) and \(S_{i}\) such that \(G=H_{i} T_{i}\) and \(H_{i} \cap T_{i} \le S_{i} \le H_{i}\).

Let us first show that \(G/L\) is \(\sigma\)-solvable for every nontrivial \(\sigma\)-primary normal subgroup \(L\) of \(G\) and, therefore, there is no \(\sigma\)-primary normal subgroup in \(G\). Indeed, suppose that \(G\) contains a minimal normal subgroup \(L\) which is a \(\sigma_{i}\)-group for some \(i\). It can readily be seen that \(\{H_{1}L/L,\dots,H_{t}L/L\}\) is a complete Hall \(\sigma\)-set of \(G/L\). Moreover, \(L \le H_{i}\) and \((|L|,|H_{j}|)=1\) for all \(j \ne i\). However, then \(H_{k}L/L\) is weakly \(K\)-\(\mathfrak{S}_{\sigma}\)-subnormal in \(G/L\) for all \(k=1,\dots,t\) by Lemma 4 (1). Then condition (iii) holds for \(G/L\), and hence \(G/L\) is \(\sigma\)-solvable due to the choice of \(G\). Thus, \(G\) is \(\sigma\)-solvable, which contradicts the choice of \(G\). Hence every minimal normal subgroup \(G\) is not \(\sigma\)-primary.

This implies that \(S^{\mathfrak{S}_{\sigma}}\ne 1\) for any nontrivial \(K\)-\(\mathfrak{S}_{\sigma}\)-subnormal subgroup \(S\) of \(G\) by Lemma 3 (5) and, therefore, \(S_{i}=1\) for all \(i\), and \(T_{1}\) is not a \(\sigma_{2}\)-group. Hence \(t > 2\), and \(T_{i}\) is a supplement to \(H_{i}\) to \(G\). Therefore, \(T_{i}\) is a Hall \( \sigma_{i}'\)-subgroup in \(G\). Then \(T_{2}\cap \cdots \cap T_{t}\) is a nonidentity Hall \(\sigma_{1}\)-subgroup of \(G\) according to [31, A, 1.6 (b)], and this subgroup is \(K\)-\(\mathfrak{S}_{\sigma}\)-subnormal in \(G\) by Lemma 2, a contradiction. Therefore, the implication (iii) \(\Rightarrow\) (i) holds.

(iv) \(\Rightarrow\) (i) Assume that this assertion fails to hold; let \(G\) be a counterexample of minimal order.

(1) If \(N\) is a minimal normal subgroup of \(G\), then \(G/N\) is a \(\sigma\)-solvable group. Thus, \(N\) is not a \(\sigma\)-primary group. Moreover, \(N\) is a unique minimal normal subgroup of \(G\), \(C_{G}(N)=1\), and \(L\le N\) for any minimal subnormal subgroup \(L\) of \(G\).

By Lemma 4 (1), the hypothesis is true for \(G/N\), which means that \(G/N\) is \(\sigma\)-solvable by the choice of \(G\). Consequently, \(N\) is not \(\sigma\)-primary.

If \(G\) contains a minimal normal subgroup \(R \ne N\), then \(G/N\) and \(G/R\) are \(\sigma\)-solvable groups, and hence it follows from the isomorphisms \(R \simeq R/1=R/(R \cap N) \simeq RN/N\) that \(R\) is a \(\sigma\)-primary group, which contradicts the fact proved above. Hence \(N\) is a unique minimal normal subgroup of \(G\) and \(C_{G}(N)=1\), since the subgroup \(N\) is non-Abelian.

It follows from [31, A, 13.4] that \(N\le N_{G}(L)\). Moreover, \(L \nleq C_{G}(N)=1\) and, therefore, \(L\le N\). Hence assertion (1) holds.

(2) If \(p\) is an arbitrary odd prime dividing \(|N|\) and \(N_{p}\) is the Sylow \(p\)-subgroup of \(N\), then \(N_{p}=N \cap G_{p}\), \(G_{p} \le N_{G}(N_{p}) \le M\), and \(G=NM\) for some Sylow \(p\)-subgroup \(G_{p}\) and some maximal subgroup \(M\) of \(G\). Hence \(p\) does not divide \(|G: M|\), and \(M_{G}=1\).

By Frattini’s argument, \(G=NN_{G}(N_{p})\). Since \(N\) is not an Abelian group, it follows that \(N_{G}(N_{p})\ne G\). Then \(N \nleq M\) for a maximal subgroup \(M\) of \(G\) containing \(N_{G}(N_{p})\). Hence \(G=NM\) and \(M_{G}=1\). Moreover, if \(N_{p} \le G_{p}\), where \(G_{p}\) is a Sylow \(p\)-subgroup of \(G\), then \(N_{p}=N\cap G_{p}\), and hence \(G_{p} \le N_{G}(N_{p}) \le M\).

(3) The intersection \(D:=M \cap N\) is not nilpotent. In particular, \(D \nleq \Phi (M)\).

Assume that \(D\) is nilpotent; let \(N_{p}\) be the Sylow \(p\)-subgroup of \(D\). Then \(N_p\) is normal in \(M\), because \(N_p\) is a characteristic subgroup of \(D\), and \(D\) is normal in \(M\). Therefore, the subgroup \(Z(J (N_{p}))\) is normal in \(M\). Since \(M_{G}=1\), it follows that \(N_{G}(Z(J(N_{p})))=M\). Then \(N_{N}(Z(J(N_{p})))=D\) is a nilpotent group. This implies that \(N\) has a normal \(p\)-complement by the Glauberman–Thompson theorem, since \(p\) is odd. However, then \(N\) is an \(p\)-group, which contradicts assertion (1). Therefore, (3) holds.

(4) \(V^{\mathfrak {S}_\sigma} \ne 1\) for every nontrivial \(K\)-\(\mathfrak{S}_{\sigma}\)-subnormal subgroup \(V\) of \(G\).

Indeed, \(V^{\mathfrak {S}_{\sigma}}=1\) implies that \(V\) is \(\sigma\)-solvable and, therefore, \(1 < V\le C_{G}(N)=1\) by Lemma 3 (5). Hence (4) holds.

(5) If \(H\) is \(K\)-\(\mathfrak{S}_{\sigma}\)-subnormal in \(G\) and is contained in \(M\), then \(H=1\). (5)

Let \(W=H^{\mathfrak{S}_{\sigma}}\) be the \(\sigma\)-solvable residual of \(H\). Then \(W\) is subnormal in \(G\) by Lemma 3 (4) and, therefore,

$$W^{G}=W^{NM}=W^{M} \le M_{G}=1$$

by [31, A, 14.3]. Hence \(W=1\). Thus, \(H\) is \(\sigma\)-solvable and, therefore, \(H=1\) by part (4).

(6) If \(H\) is a weakly \(K\)-\(\mathfrak {S}_{\sigma}\)-subnormal subgroup of \(G\) and is contained in \(M\), then \(G=HT\) and \(H \cap T=1\) for some \(K\)-\(\mathfrak {S}_{\sigma}\)-subnormal subgroup \(T\) of \(G\) (this follows from part (5) and from the definition of weakly \(K\)-\(\mathfrak {S}_{\sigma}\)-subnormal subgroup).

(7) \(M\) is not weakly \(K\)-\(\mathfrak{S}_{\sigma}\)-subnormal in \(G\). In particular, \(M\) is \(\sigma\)-solvable.

Suppose that \(M\) is weakly \(K\)-\(\mathfrak{S}_{\sigma}\)-subnormal in \(G\). Then \(G=MT\) and \(M \cap T=1\) for some \(K\)-\(\mathfrak{S}_{\sigma}\)-subnormal subgroup \(T\) of \(G\) by assertion (6). Moreover, \(W:=T^{\mathfrak {S}_{\sigma}} \ne 1\) according to part (4), and \(W\) is a subnormal subgroup of \(G\) according to Lemma 3 (4).

Let now \(L\) be a minimal subnormal subgroup of \(G\) contained in \(W\). Then \(L\) is a minimal normal subgroup of \(N\) by part (1). Hence \(p\) divides \(|L|\), where \(|L|\) divides \(|T|=|G: M|\). However, \(p\) does not divide \(|G: M|\) by part (2). This contradiction completes the proof of the first part of (7). Then, in any maximal chain

$$\cdots < M_{3} < M_{2} < M_{1}=M < G$$

of \(G\), one of the subgroups \(M_{3}\) and \(M_{2}\) is weakly \(K\)-\(\mathfrak{S}_{\sigma}\)-subnormal in \(G\) by assumption. Hence this subgroup is weakly \(K\)-\(\mathfrak{S}_{\sigma}\)-subnormal in \(M\) by Lemma 4 (3). Thus, the conjecture holds for \(M\), and thus \(M\) is \(\sigma\)-solvable by the choice of \(G\).

(8) The relation \(N < G\) holds.

Suppose that \(N=G\) is a non-Abelian simple group; let \(H\) be an arbitrary proper \(K\)-\(\mathfrak{S}_{\sigma}\)-subnormal subgroup of \(G\). Suppose that \(H\ne 1\).

Then \(G\) has a proper nonidentity subgroup \(V\) such what \(H \le V\) and either \(V\) is normal in \(G\) or \(G/V_{G}\) is a \(\sigma\)-solvable group. However, the first condition is impossible, since the group \(G\) is simple. Hence \(V_{G}=1\), and then \(G/V_{G}=G/1\simeq G\) is a \(\sigma\)-solvable group, which contradicts the choice of \(G\). Thus, any proper \(K\)-\(\mathfrak{S}_{\sigma}\)-subnormal subgroup of \(G\) is trivial.

Let \(Q\) be a Sylow \(q\)-subgroup of \(G\), where \(q\) is the least prime dividing \(|G|\), and let \(L\) be a maximal subgroup of \(G\) containing \(Q\). Then, taking into account [32, IV, 2.8], we obtain \(|Q| >q\). Let \(V\) be a maximal subgroup of \(Q\) and \(S\) be a maximal subgroup of \(V\). Then there is a 3-maximal subgroup \(1 < W\) of \(G\) such that \(W\le Q\). Indeed, if \(S\ne 1\), then this is obvious. On the other hand, if \(S=1\), then \(Q\) is an Abelian group and, therefore, \(Q < L\) by [32, IV, 7.4]. Therefore, in this case, there exists a 3-maximal subgroup \(W\) of \(G\) such that \(V \le W \le Q\).

Due to condition (iv), \(G\) has a weakly \(K\)-\(\mathfrak{S}_{\sigma}\)-subnormal subgroup \(U\) such what \(W\le U < G\). Hence \(G\) contains \(K\)-\(\mathfrak{S}_{\sigma}\)-subnormal subgroups \(T\) and \(R\) such that \(G=UT\) and \(U \cap T \le R \le U\). Then \(T=G\), and thus \(U=U \cap T \le R \le U\).

Hence \(U=R\) is \(K\)-\(\mathfrak{S}_{\sigma}\)-subnormal in \(G\). Then \(U=1\), a contradiction. Thus, assertion (8) holds.

(9) If \(G_{p} \le V \le M\), then \(V\) is not weakly \(K\)-\(\mathfrak{S}_{\sigma}\)-subnormal in \(G\).

Assume that \(V\) is weakly \(K\)-\(\mathfrak{S}_{\sigma}\)-subnormal in \(G\). Then \(G=VT\) and \(V \cap T=1\) for some \(K\)-\(\mathfrak{S}_{\sigma}\)-subnormal subgroup \(T\) of \(G\) by assertion (6). Then \(1 < T < G\), and the subgroup \(T^{\mathfrak {S}_{\sigma}}\) is nontrivial by part (4). Moreover, this subgroup is subnormal in \(G\) by Lemma 3 (4). Hence \(Z \le T\) for a minimal subnormal subgroup \(Z\) of \(G\) contained in \(T^{\mathfrak {S}_{\sigma}}\). Then \(Z\) is a \(p'\)-group, because \(V \cap T=1\) and \(G_{p} \le V\). On the other hand, \(Z\) is a minimal normal subgroup of \(N\) by part (1) and, therefore, \(p\) divides \(|Z|\). This contradiction completes the proof of (9).

(10) The group \(M=D \rtimes L\) is solvable, where \(L\) is a group of prime order.

By [32, IV, 7.4], \(G_{p} \le L\) for some maximal subgroup \(L\) of \(M\), since \(p > 2\) and \(G\) is unsolvable. Let us first show that \(G_{p}=L\). Indeed, suppose that \(G_{p} \le V\) for some maximal subgroup \(V\) of \(L\). Then, in every maximal chain

$$\cdots < V=M_{i} < \cdots <M_{3} < M_{2} < M_{1}=M < M_{0}=G$$

of \(G\) that contains \(V\) below \(M_{3}\), all subgroups \(M_{3}\), \(M_{2}\), and \(M_{1}\) are not weakly \(K\)-\(\mathfrak{S}_{\sigma}\)-subnormal in \(G\) by part (9), which contradicts the condition. Hence \(G_{p}=L\) is a maximal subgroup of \(M\) and, therefore, \(M\) is solvable according to [32, IV, 7.4]. If \(G_{p} \le D\), then \(D=M \cap N=M\), since \(G_{p} \ne D\) by part (2), whence it follows that \(N=G\), which contradicts part (8). Hence \(G_{p} \nleq D\), and thus \(DL=M\). Therefore, \(D \cap L=1\) in the case of \(|G_{p}|=p\), whence it follows that \(M=D \rtimes L\), and thus assertion (10) holds because \(M\) is solvable.

Finally, suppose that \(|G_{p}| >p\). First of all, note what \(L\) is not weakly \(\sigma \)-subnormal in \(G\) by part (9). However, on the other hand, \(M\) is also not weakly \(K\)-\(\mathfrak{S}_{\sigma}\)-subnormal in \(G\) by (7). Therefore, every maximal subgroup of \(L=G_{p}\) is weakly \(K\)-\(\mathfrak {S}_{\sigma}\)-subnormal in \(G\).

It follows from \(G_{p} \nleq D\) that \(N_{p}=N \cap G_{p} \le V\) for some maximal subgroups \(V\) of \(G_{p}\). Since \(V\) is weakly \(K\)-\(\mathfrak{S}_{\sigma}\)-subnormal in \(G\), it follows that \(V\) has a \(K\)-\(\mathfrak{S}_{\sigma}\)-subnormal complement \(T\) in \(G\) by part (7). Hence \(T \cap N_{p}= 1\). Lemma 3 (4) and part (4) imply that \(Z\le T^{\mathfrak{S}_{\sigma}}\le T\) for some minimal subnormal subgroup \(Z\) of \(G\). Then \(Z \cap N_{p}=1\) and, therefore, \(Z \nleq N\), which contradicts part (1). Therefore, assertion (10) holds.

A concluding contradiction for (iv) \(\Rightarrow\) (i). It follows from part (10) that \(|D|\) is a power of a prime, which contradicts part (2). Therefore, the implication (iv) \(\Rightarrow\) (i) holds.

(v) \(\Rightarrow\) (i) Suppose that this assertion fails to hold; let \(G\) be a counterexample of minimal order.

First, note that, if \(H\le A\le G\), where \(H\) is not \(K\)-\(\mathfrak{S}_{\sigma}\)-subnormal in \(A\), then \(H\) is not \(K\)-\(\mathfrak{S}_{\sigma}\)-subnormal in \(G\) by Lemma 3 (1). Hence \(i_{K,\mathfrak{S}_{\sigma}}(A)\le i_{K,\mathfrak{S}_{\sigma}}(G)\) for every subgroup \(A\) of \(G\). Moreover, if \(H/N\le G/N\), where \(H/N\) is not \(K\)-\(\mathfrak{S}_{\sigma}\)-subnormal in \(G/N\), then \(H\) is not \(K\)-\(\mathfrak{S}_{\sigma}\)-subnormal in \(G\) by Lemma 3 (2). Hence \(i_{K,\mathfrak{S}_{\sigma}}(G/N)\le i_{K,\mathfrak{S}_{\sigma}}(G)\).

(1) If \(N\) is a nonidentity normal subgroup of \(G\) such that \(|\sigma(G/N)|=|\sigma(G)|\), then \(G/N\) is \(\sigma\)-solvable. Moreover, if \(L\) is a proper subgroup of \(G\) such that \(|\sigma(L)|=|\sigma(G)|\), then \(L\) is \(\sigma\)-solvable.

Since

$$i_{K,\mathfrak{S}_{\sigma}}(G/N)\le i_{K,\mathfrak{S}_{\sigma}}(G)\le 2|\sigma(G)|=2|\sigma(G/N)|,$$

it follows that the conjecture holds for \(G/N\) and, therefore, \(G/N\) is \(\sigma\)-solvable by the choice of \(G\). Similarly, it follows from \(i_{K,\mathfrak{S}_{\sigma}}(L) \le i_{K,\mathfrak{S}_{\sigma}}(G)\) that \(L\) is \(\sigma\)-solvable.

(2) The equality \(O_{\sigma_{i}}(G)=1\) holds for every \(\sigma_i\in \sigma(G)\).

Suppose that \(O:= O_{\sigma_i} (G)\ne 1\) for some \(i\). Let us first show that \(O\nleq \Phi(G)\). Indeed, if \(O \le \Phi(G)\), then \(\pi(G/O)=\pi(G)\) and, therefore, \(\sigma(G/O)=\sigma(G)\), and thus \(G/O\) is \(\sigma\)-solvable by part (1). Consequently, \(G\) is \(\sigma\)-solvable; a contradiction. Hence \(O\nleq \Phi(G)\), and thus there is a maximal subgroup \(M\) of \(G\) such that \(G=OM\). Then \(M\) is not a \(\sigma\)-solvable group and, therefore, \(|\sigma(M)|\ne| \sigma(G)|\) by part (1). On the other hand, \(|G: M|=|O: Q\cap M|\) is a \(\sigma_{i}\)-number and, therefore, \(|\sigma(M)|=|\sigma(G)|-1\). This implies that \(M\cap O=1\) and, therefore, \(O\) is a Hall \(\sigma_i\)-subgroup of \(G\).

The inequality \(i_{\sigma}(M)\ge 2|\sigma(M)|+1\) holds by the choice of \(G\). Let \(i_{K,\mathfrak{S}_{\sigma}}(M)=k\). If \(k=0\), then all subgroups of \(M\) are \(K\)-\(\mathfrak{S}_{\sigma}\)-subnormal in \(M\) and, therefore, \(M\) is \(\sigma\)-solvable by implication (ii) \(\Rightarrow\) (i). Thus, \(k\ne 0\). Let \(T_1,\dots,T_k\) be representatives of the classes of non-\(K\)-\(\mathfrak{S}_{\sigma}\)-subnormal subgroups of \(M\) of equal order. Since \(T_j=T_j(O\cap M)=OT_j\cap M\) is not a \(K\)-\(\mathfrak{S}_{\sigma}\)-subnormal subgroup of \(M\), it follows that \(OT_j\) is not \(K\)-\(\mathfrak{S}_{\sigma}\)-subnormal in \(G\) by Lemma 3 (1). Moreover, since \(M\cap O=1\) and \(|T_{j_1}|\ne |T_{j_2}|\) for \(j_1\ne j_2\), it follows that \(|OT_{j_1}|\ne |OT_{j_2}|\) for all \(j_1\ne j_2\).

Thus, taking into account the inequality \(i_{K,\mathfrak{S}_{\sigma}}(G)\ge i_{K,\mathfrak{S}_{\sigma}}(M)\) and counting the subgroups \(OT_i\), we obtain

$$i_{K,\mathfrak{S}_{\sigma}}(G)\ge 2i_{K,\mathfrak{S}_{\sigma}}(M)\ge 2(2|\sigma (M)|+1).$$

However, since \(2\le |\sigma (M)|=|\sigma (G)|-1\), it follows that

$$i_{K,\mathfrak{S}_{\sigma}}(G)\ge 2(|\sigma(G)|-1)+ 2|\sigma(M)|+2\ge 2|\sigma (G)|+4.$$

This contradiction completes the proof of assertion (2).

(3) If \(H\) is a nonidentity \(\sigma\)-solvable subgroup of \(G\), then \(H\) is not \(K\)-\(\mathfrak {S}_{\sigma}\)-subnormal in \(G\) (this follows from part (2) and Lemma 3 (5)).

(4) The equality \(O^{\sigma_i}(G)=G\) holds for every \(\sigma_i\in\sigma(G)\).

Suppose that \(O=O^{\sigma_i}(G)< G\) for some \(i\). Since \(G/O\) is a \(\sigma_i\)-group, it follows that \(G/O\) is \(\sigma\)-solvable. Hence

$$i_{K,\mathfrak{S}_{\sigma}}(O)\ge 2|\sigma(O)|+1$$

by the choice of \(G\).

By assertion (1), \(|\sigma(O)|=|\sigma(G)|-1\) and, therefore, \(O\) is a Hall \(\sigma_i'\)-subgroup of \(G\). Therefore, by the Schur–Zassenhaus theorem, there is a complement \(H\) for \(O\) in \(G\), \(H\) is a Hall \(\sigma_i\)-subgroup of \(G\), and every Hall \(\sigma_i\)-subgroup of \(G\) is conjugate to \(H\).

Note that \(H\) is simple. Indeed, suppose what \(N\ne 1\) is a proper normal subgroup of \(H\). Then \(ON\) is a proper normal subgroup of \(G\), and \(|\sigma(ON)|=|\sigma(G)|\). Hence \(ON\) is \(\sigma\)-solvable by assertion (1), which implies that \(G\) is \(\sigma\)-solvable, since \(G/ON\) is a \(\sigma_i\)-group, a contradiction. Thus, \(H\) is a simple group.

Since \(O\) and \(H\) are Hall subgroups of \(G\), it follows that either \(O\) or \(H\) is solvable by the Feit–Thompson theorem on the solvability of groups of odd order. However, if \(O\) is solvable, then \(O\) is \(\sigma\)-solvable and, therefore, \(G\) is \(\sigma\)-solvable. Hence \(H\) is solvable, and thus \(|H|=q\) for some \(q\in \pi(G)\). Consequently, \(H\) is not \(K\)-\(\mathfrak{S}_{\sigma}\)-subnormal in \(G\) by part (3). Moreover, every nonidentity Sylow subgroup of \(O\) is not \(K\)-\(\mathfrak{S}_{\sigma}\)-subnormal in \(O\). Let \(P\) be some Sylow \(p\)-subgroup of \(O\), \(p\in \pi(O)\). According to Frattini’s argument, \(G=ON_G(P)\). Since \(q\) does not divide \(|O|\), it follows that \(q\) divides \(|N_G(P)|\). Hence there is an \(x\in G\) such that \(H^x\le N_G(P)\) and, therefore, \(H^{x}P\) is a subgroup of \(G\). Since

$$P=P(H^{x}\cap O)=H^{x}P\cap O$$

is not \(K\)-\(\mathfrak{S}_{\sigma}\)-subnormal in \(O\), it follows that \(H^{x}P\) is not \(K\)-\(\mathfrak{S}_{\sigma}\)-subnormal in \(G\) by Lemma 3 (1). We also note that \(|\pi (O)|\ge 2\) by the \(p^aq^b\)-Burnside theorem. Therefore, the number of subgroups of the form \(H^{x}P\) is not less than 2. Thus, taking into account that

$$i_{K,\mathfrak{S}_{\sigma}}(G)\ge i_{K,\mathfrak{S}_{\sigma}}(O)\ge 2|\sigma (O)|+1=2(|\sigma (G)|-1)+1=2(|\sigma (G)|-1$$

and taking into account the subgroups \(H^{x}P\) and \(H\), we obtain

$$i_{K, \mathfrak{S}_{\sigma}}(G)\ge i_{K,\mathfrak{S}_{\sigma}}(O)+2+1\ge 2|\sigma (G)|+2.$$

This contradiction completes the proof of assertion (4).

A concluding contradiction for (v) \(\Rightarrow\) (i). Since \(G\) is unsolvable, it follows that \(G\) has a noncyclic Sylow subgroup \(P\) by [32, VI, 10.3]. Then a maximal subgroup \(V\) of \(P\) is not identity and, therefore, \(V\) is not \(K\)-\(\mathfrak{S}_{\sigma}\)-subnormal in \(G\) by part (3). Moreover, every nonidentity Sylow subgroup of \(G\) is not \(K\)-\(\mathfrak{S}_{\sigma}\)-subnormal in \(G\). Therefore, \(m \ge |\pi(G)|+1\) for the number \(m\) of all classes of non-\(K\)-\(\mathfrak{S}_{\sigma}\)-subnormal primary subgroups of equal order.

By assertion (4), \(G\) is not \(p\)-nilpotent for any \(p\in\pi(G)\). Therefore, taking into account [32, III, 5.2], we see that there is a \(p\)-closed Schmidt subgroup \(E\) of \(G\) with \(p\in \pi(E)\) for every \(p\in \pi(G)\).

Now let \(p> q\) be distinct prime divisors of \(|G|\). Let \(V\) be a \(p\)-closed Schmidt subgroup with \(p\in \pi(V)\), and let \(W\) be a \(q\)-closed Schmidt subgroup with \(q\in \pi(W)\). By [32, IV, 5.4], \(V=V_p\rtimes V_r\) and \(W=W_q\rtimes W_t\), where \(V_p\) is a Sylow \(p\)-subgroup and \(V_r\) is a cyclic Sylow \(r\)-subgroup of \(V\) (\(r\ne p\)); \(W_q\) is a Sylow \(q\)-subgroup and \(W_t\) is a cyclic Sylow \(t\)-subgroup of \(W\) (\(t\ne q\)). By part (4), \(V\) and \(W\) are not \(K\)-\(\mathfrak{S}_{\sigma}\)-subnormal in \(G\). If \(|V|\ne |W|\), then \(V\) and \(W\) are representatives of two distinct classes of non-\(K\)-\(\mathfrak{S}_{\sigma}\)-subnormal subgroups in \(G\) of equal order.

Now suppose that \(|V|=|W|\). In this case, \(r=q\) and \(t=p\). Let \(T\) be a maximal subgroup of \(V\) such that \(V_p\le T\). If \(V_p=T\), then \(|V_q|=q\), and thus, taking into account that \(|V|=|W|\), we see that \(|W_q|=q\). Hence \(W\) is supersolvable. As is well known, every supersolvable group is Ore dispersive. Therefore, \(W\) is Ore dispersive, which implies that \(q>p\), a contradiction. Thus, \(V_p <T\). Moreover, \(T\) is not \(K\)-\(\mathfrak{S}_{\sigma}\)-subnormal in \(G\) by part (4), and \(T\) is not a primary group. Thus, for every \(p\in \pi(G)\), there is a class of non-\(K\)-\(\mathfrak{S}_{\sigma}\)-subnormal subgroups in \(G\) of equal order, and each representative of any of these classes is not a primary group. It is also clear that \(r \ge\pi(G)|\) for the number \(r\) of such classes. Hence

$$i_{K,\mathfrak{S}_{\sigma}}(G)\ge |\pi(G)|+1+|\pi(G)| \ge 2|\sigma (G)|+1.$$

This contradiction completes the proof of the implication (v) \(\Rightarrow\) (i).

This completes the proof of the theorem.